How to Compute Base 3 Floating Point Expansion of 1/4?

Click For Summary
To compute the base 3 floating point expansion of 1/4, one can use a long division method similar to that used in base 10. The fraction 1/4 translates to 1/11 in base 3. The process begins by dividing 11 into 1, which yields zero, followed by bringing down zeros until reaching 100 in base 3. The first nonzero digit in the expansion can then be determined by continuing the division process. It is important to note that the correct terminology is "base 3 floating point" expansion, not "decimal."
hedipaldi
Messages
209
Reaction score
0

Homework Statement



How to compute the base 3 decimal expansion of 1/4?

Homework Equations





The Attempt at a Solution


I tried sums of geometric sequences,but i need a clue for the computation.
 
Physics news on Phys.org
hedipaldi said:

Homework Statement



How to compute the base 3 decimal expansion of 1/4?

Homework Equations


The Attempt at a Solution


I tried sums of geometric sequences,but i need a clue for the computation.

You know how to find 1/4=0.25 in base 10 using long division, right? In base 3 that's 1/11. Do the same process, just do all the arithmetic in base 3.
 
So far i already got.How to proceed?
 
hedipaldi said:
So far i already got.How to proceed?

Just do it. 11 goes into 1 zero times. So you write down 0. and 'bring down' a 0. Now you've got 11 into 10. Still zero times. Answer so far 0.0, bring down another 0. Now you've got 11 into 100 in base 3. So what's the first nonzero digit in the expansion? Multiply subtract and keep going. I'm not really sure how else to explain it in words.
 
Thank's a lot.
 
Just as a nitpicking point, it is not correctly called base 3 DECIMAL expansion since "decimal" is specifically for the base 10 system. It is called "base 3 floating point" expansion.
 
Question: A clock's minute hand has length 4 and its hour hand has length 3. What is the distance between the tips at the moment when it is increasing most rapidly?(Putnam Exam Question) Answer: Making assumption that both the hands moves at constant angular velocities, the answer is ## \sqrt{7} .## But don't you think this assumption is somewhat doubtful and wrong?

Similar threads

  • · Replies 7 ·
Replies
7
Views
2K
Replies
32
Views
3K
  • · Replies 7 ·
Replies
7
Views
2K
  • · Replies 5 ·
Replies
5
Views
3K
  • · Replies 1 ·
Replies
1
Views
2K
Replies
12
Views
2K
  • · Replies 20 ·
Replies
20
Views
3K
  • · Replies 14 ·
Replies
14
Views
2K
  • · Replies 1 ·
Replies
1
Views
2K
Replies
6
Views
2K